Solve for q.

0.48 + 5.7q = 6.3q

q =

Answers

Answer 1

Answer:

[tex] \boxed{q = 0.8} [/tex]

Step-by-step explanation:

[tex] = > 0.48 + 5.7q = 6.3q \\ \\ = > 6.3q = 5.7q + 0.48 \\ \\ = > 6.3q - 5.7q = 0.48 \\ \\ = > 0.6q = 0.48 \\ \\ = > q = \frac{0.48}{0.6} \\ \\ = > q = 0.8[/tex]

Answer 2

Answer:

q = 0.8

Step-by-step explanation:

isolate the variable and solve


Related Questions

The product of two positive numbers is 1,800. One number is twice the other number. The equation to determine the two numbers, with x representing the value of the lesser number, is x(2x) = 1,800. What is the value of the lesser number?

Answers

Answer:

600

Step-by-step explanation:

x+2x=1800

3x=1800

x=600

Answer:

- 30

Step-by-step explanation:

The 2 numbers are x and 2x and thir product is 1800, that is

x × 2x = 1800

2x² = 1800 ( divide both sides by 2 )

x² = 900 ( take the square root of both sides )

x = ± [tex]\sqrt{900}[/tex] = ± 30

The 2 numbers are 30 and - 30 with - 30 being the lesser number

solve for k.

6(3k + 15) − 16k = 7k

k =

Answers

Answer:

k = 18

Step-by-step explanation:

6(3k + 15) − 16k = 7k

Distribute

18k +90 - 16k = 7k

Combine like terms

2k + 90 = 7k

Subtract 2k from each side

2k+90-2k = 7k -2k

90 = 5k

Divide by 5

90/5 = 5k/5

18 = k

Answer:

[tex]k = 18[/tex]

Step-by-step explanation:

[tex]6(3k + 15)- 16k = 7k \\ 18k + 90 - 16k = 7k \\ 18k - 16k - 7k = - 90 \\ - 5k = - 90 \\ \frac{ - 5k}{ - 5} = \frac{ - 90}{ - 5} \\ k = 18[/tex]

hope this helps

brainliest appreciated

good luck! have a nice day!

PLeAse HELP me! WIll GIVE Brainliest!!

Write an equation of the circle with center (-2,3) and radius 5.

Answers

Answer:

The formula is: [tex](x+2)^2+(y-3)^2=25[/tex]

Step-by-step explanation:

Recall the formula for the equation of a circle with center at the point  [tex](x_0,y_0)[/tex]of radius "R' as:

[tex](x-x_0)^2+(y-y_0)^2=R^2[/tex]

Therefore for your case, the formula becomes:

[tex](x-x_0)^2+(y-y_0)^2=R^2\\(x-(-2))^2+(y-3)^2=5^2\\(x+2)^2+(y-3)^2=25[/tex]

Please I want brainliest lol!

Please answer correctly !!!!! Will mark brainliest !!!!!!!!!!!!

Answers

Answer:

Up by 3 units, right by 9 units.

Step-by-step explanation:

The subtraction in (x-9)^2 makes the function shift to the right, adding would make it shift left! Adding numbers to the equation would make the function go up, and subtracting would make it go down. Hope this helps :)

How do you complete the square of h(x)= x^2 + 6x + 7

Answers

Answer:

[tex] (x + 3)^{2} - 2[/tex]

Step-by-step explanation:

[tex]h(x)= x^2 + 6x + 7 \\ = x^2 + 2.3.x + {3}^{2} - {3}^{2} + 7 \\ = (x + 3)^{2} - 9 + 7 \\ \purple { \bold{h(x)= (x + 3)^{2} - 2 }}[/tex]

Is the graph a linear,nonlinear function,or relation

Answers

Answer:

Step-by-step explanation:

nonlinear ig?

it is non linearrrrrrrrrrrr

Find x if f(x)=2x+7 and f(x)=-1

Answers

Answer: -4

Step-by-step explanation:idk

Answer:

x= -4

Step-by-step explanation:

Take the two functions to equal to eachother, bring 7 to the right side of the equal sign which will now be negative and minus it with -1 to get -8 . Then divide 2 both sides to get x= -4.

help me solve this please .

Answers

Answer:

[tex]\boxed{\sf \ \ \ d=6 \ \ \ }[/tex]

Step-by-step explanation:

we know (thanks to Pythagoras) that

[tex]d = \sqrt{5^2+3^2}= \sqrt{25+9}= \sqrt{36}=6[/tex]

The truth table represents statements p, q, and r.

p q r p ∧ q p ∧ r
A T T T T T
B T T F T F
C T F T F T
D T F F F F
E F T T F F
F F T F F F
G F F T F F
H F F F F F
Which rows represent when (p ∧ q) ∨ (p ∧ r) is true?

A and B
A, B, and C
B and E

Answers

Answer:

A, B, and C

Step-by-step explanation:

EDGE unit test 2020

The rows that stands for when (p ∧ q) ∨ (p ∧ r) is true is row A, B, and C. Check more about Truth table below

What is a truth table?

The truth table is known to be a table that depicts the real value of any kind of compound statement.

Note that for every truth-value of a statements there is a similar table depicting the value of the output for all of the value of all of the  input and as such, rows that stands for when (p ∧ q) ∨ (p ∧ r) is true is row A, B, and C.

Learn more about Truth table from

https://brainly.com/question/14458200

#SPJ2

Look at the graph below. Which of the following best represents the slope of the line? A. -3 B. - 1 3 C. 1 3 D. 3

Answers

Answer:

3

Step-by-step explanation:

The slope of the given line is [tex]\frac{2}{3}[/tex].

What is the slope of a line passing through two points?

If a line passes through two points (x₁, y₁) and (x₂, y₂) respectively, then slope of the line is [tex]m = \frac{y_{2} - y_{1}}{x_{2} - x_{1}}[/tex]

Here, the given line passes through points (0, 6) and (-9, 0).

Therefore, the slope of this line is (m)

[tex]= \frac{0 - 6}{-9 - 0}\\= \frac{6}{9} \\= \frac{2}{3}[/tex]

Learn more about the slope of a line here:

https://brainly.com/question/19549883

#SPJ2

The mean and standard deviation for the heights of men in the U.S. are 70 inches and 4 inches respectively and are normally distributed. Based on this information, which of the following represents the percent of men whose heights falls between 58 inches and 66 inches to the nearest whole percent?

A) 2%
B) 16%
C) 34%
D) 68%

Answers

Answer:

B) 16%

Step-by-step explanation:

First, we need to standardize 58 and 66 inches using the following equation:

[tex]z=\frac{x-m}{s}[/tex]

Where m is the mean and s is the standard deviation, so 58 and 66 are equivalent to:

[tex]z=\frac{58-70}{4}=-3\\z=\frac{66-70}{4}=-1\\[/tex]

Then, the percent of men whose heights falls between 58 inches and 66 inches is calculated as:

P(58<z<66) = P(-3<z<-1)

So, using the normal table, we get:

P(-3<z<-1) = P(z<-1) - P(z<-3)

P(-3<z<-1) = 0.1587 - 0.0013

P(-3<z<-1) = 0.1574

Finally, 0.1574 rounded to the nearest whole percent is equal to 16%

Graph y < x^2 - 3. please help

Answers

Answer:

Step-by-step explanation:

the shaded area is a solution.

FIND THE VALUE OF X FOR THIS MATH QUESTION

Answers

Answer:

x = 2

Step-by-step explanation:

Since the central angles are congruent, and all radii are congruent, the triangles are congruent. By CPCTC x = 2.

What is the length of the diagonal, d, of the rectangular prism shown below?

Answers

Answer:

The answer is 8.3

Step-by-step explanation:

(I'm not that good at explaining things so bear with me please. I checked on Khan Academy so it's right)

I split the box on the right diagonally and got the equation

[tex]4^{2}[/tex] + [tex]2^{2}[/tex] = [tex]x^{2}[/tex]

=16 + 4 =[tex]\sqrt{20}[/tex]

≈4.5

The new triangle will help solve what d is

[tex]4.5^{2}[/tex] + [tex]7^{2}[/tex] = [tex]d^{2}[/tex]

=20.25 +49 = [tex]\sqrt{69.25}[/tex]

≈8.3

The diagonal of the rectangular prism shown is 8.3.

What is diagonal?

A polygonal is a closed, two-dimensional structure that is flat or plane and seems to have excellent dimensional.

A diagonal is a line segment that joins 2 opposing polygonal vertices.

In another word, a diagonal is an inclined line that connects two orthogonal lines to become a complete triangle.

Diagonal is the use for the idea of the length of sides, for example, if you want to measure the height of a tower knowing diagonal will surely help to do this.

Given a rectangular prism

Let x is the diagonal of the sides 7 and 4 then

By Pythagoras theorem

4² + 7² = x²

x² = 65

x=8.06

Now in the right-angle triangle of sides d,x and 2

d² = 65 + 4

D = √(69)

d  = 8.30 hence, the correct answer is 8.30.

For more about the diagonal

https://brainly.com/question/14735698

#SPJ5

During the Hari Keusahawanan in a school, 800 booklets of coupons were sold. The price of each booklet of coupons was RM30 and RM50 respectively. The total amount collected was RM30 000. How many booklets of RM30 and RM50 coupons were sold?

(**please use the simultaneous linear equations in two variables to solve this question) ​

Answers

Answer:

hope this is correct

you can always _____ two similar triangles so that they become congruent.

Answers

Answer:

the answer is have to be equal.

Step-by-step explanation:

you can always equal two similar triangles so that they become congruent.

In the ratio of the length of their sides is equal to one, they are congruent. So, all congruent triangles have to be similar but all similar triangles need not be congruent......the ratio of corresponding sides,AB/A'B'=BC/B'C'=CA/C'A'. If the two triangles are congruent AB/A'B'=BC/B'C'=CA/C'A'=1

I am glad to help you!!!!

Complete the statements.
A triangular prism has__bases and___lateral faces
A triangular pyramid has____
bases and___lateral faces

Answers

Answer:

A triangular prism has two bases and three lateral faces.

A triangular pyramid has one  base and three lateral faces.

Step-by-step explanation:

A triangular prism is a solid geometrical shape. The base is triangular in shape. There are two bases that are triangular. Other lateral faces are rectangular in shape. They are three in number.

A triangular pyramid is also a solid geometrical shape. The base is a polygon. The lateral faces are triangular in shape and are three in number. The lateral faces are congruent.

Answer:

So im to lazy to right the answer so just look above me so hows it going?

Step-by-step explanation:

Can someone please help ?!

Answers

Answer:

4 * pi

Step-by-step explanation:

Given r = 10

Formula for the circumference of this full circle is:

2 * r * pi

2 * 10 * pi = 20 * pi

Given one (biggest) part = 16 * pi of this circle.

The other (smaller) part must therefore be:

20 * pi - 16 * pi

4 * pi

Suppose a car rental company charges $22.50 plus 10 cents per mile write a function rule that models this situation

Answers

Answer:

C(m) = 22.50 + .10m

Step-by-step explanation:

Cost = fee + mileage

Cost = 22.50 + .10m  where m is the number of miles

C(m) = 22.50 + .10m

The area of a rectangle is 112 units squared.
The length is 7 and the width is the square root of "x".
What would be the value of "x"?
What is the width of the rectangle?

Answers

Answer:

Step-by-step explanation:

The area for a rectangle is A = LW. We know the area and we also know the length. We are solving for the width.

[tex]112=7\sqrt{x}[/tex] and

[tex]16=\sqrt{x}[/tex]

Siince the width is given as the square root of x, and the square root of x is equal to 16, then the width is 16. Now to find the value of x from the last statement in our equation. If

[tex]16=\sqrt{x}[/tex] and we want to solve for x, we will square both sides, since squaring undoes a square root. Since this is an equation, we have to square both sides.

256 = x

The area of the rectangle is 112 units and the width is 16 units

What is the Area of a Rectangle?

The area of the rectangle is given by the product of the length of the rectangle and the width of the rectangle

Area of Rectangle = Length x Width

Given data ,

Let the area of the rectangle be represented as A

Now , the value of A is

A = 112 units

Let the length of the rectangle be L = 7 units

Let the width of the rectangle be W = √x units

Now , Area of Rectangle = Length x Width

Substituting the values in the equation , we get

Area of the rectangle = 7 √x

7√x = 112

Divide by 7 on both sides of the equation , we get

√x = 16

Taking the square on both sides of the equation , we get

x = 16²

x = 256

Therefore , the value of x is 256

Hence , the width of the rectangle is √x = 16 units

To learn more about area of rectangle click :

https://brainly.com/question/15225905

#SPJ2

simplify (3x+5)+(2x-9)-(4x+3)

Answers

Answer: X-7

Step-by-step explanation:

I put it into Tiger Algebra Calculator and that is what I got. Great tool btw I would recommend.

Answer:

3x+2x-4x+5-9+3

5x-4x+8-9

x-1

hope it helps

how to find a side length in a triangle with all the degrees and one side length

Answers

Answer:Case I

When we know two sides of the right triangle, in which case, we will use the Pythagorean theorem

Case II

When we know 1 side and 1 angle of the right triangle, in which case, we will use good old sohcahtoa

Step-by-step explanation:

Solve for y.

–0.8y + 5.49 = –19.86 − 2.3y

y =

Answers

Answer:

y = -16.9

Step-by-step explanation:

→Add 0.8y to both sides:

-0.8y + 5.49 = -19.86 - 2.3y

5.49 = -19.86 - 1.5y

→Add 19.86 to both sides:

25.35 = -1.5y

→Divide both sides by -1.5:

-16.9 = y

Answer:

[tex] \boxed{y = - 16.9} [/tex]

Step-by-step explanation:

[tex] = > - 0.8y + 5.49 = - 19.86 - 2.3y \\ \\ = > - 0.8y + 2.3y + 5.49 = - 19.86 \\ \\ = > 1.5y + 5.49 = - 19.86 \\ \\ = > 1.5y = - 19.86 - 5.49 \\ \\ = > 1.5y = - 25.35 \\ \\ = > y = - \frac{25.35}{1.5} \\ \\ = > y = - 16.9[/tex]

In quadrilateral QRST, m is 68°, m is (3x + 40)°, and m is (5x − 52)°. What are the measures of , , and ? Write the numerical values in that order with the measures separated by commas.

Answers

Answer:

m∠R = 112°, m∠S = 112°, m∠T = 68°

Step-by-step explanation:

In a cyclic quadrilateral QRST,

m∠R + m∠T = 180° [Since opposite angles of a cyclic quadrilateral measure 180°]

(3x + 40)° + (5x - 52)° = 180°

8x - 12 = 180

8x = 180 + 12

8x = 192

x = 24

m∠R = (3x + 40) = 3×24 + 40

                          = 72 + 40

                          = 112°

m∠T = 180° - m∠R

        = 180° - 112°

        = 68°

Since m∠Q = 68°

m∠S = 180° - m∠Q

        = 180° - 68°

        = 112°

Therefore, m∠R = 112°, m∠S = 112°, m∠T = 68°

Answer:

112, 112, 68

Step-by-step explanation:

It took me over 15 min to figure out what the answer was. When answering it, you have to be very specific.

A 14​-foot ladder is placed against a vertical wall of a​ building, with the bottom of the ladder standing on level ground 9 feet from the base of the building. How high up the wall does the ladder​ reach?

Answers

Answer:

10.7 feet

Step-by-step explanation:

The ladder, the ground and the wall form the shape of a right angled triangle as shown in the image below.

The hypotenuse of the triangle is 14 feet (length of ladder)

The base of the triangle is 9 feet long (the distance from the base of the ladder to the wall)

We need to find the height of the triangle. We can apply Pythagoras rule:

[tex]hyp^2 = a^2 + b^2[/tex]

where hyp = hypotenuse

a = base of the triangle

b = height of the triangle

Therefore:

[tex]14^2 = 9^2 + b^2\\\\196 = 81 + b^2\\\\b^2 = 196 - 81 = 115\\\\b = \sqrt{115} \\\\b = 10.7 feet[/tex]

The wall reaches 10.7 feet high.

In ΔWXY, the measure of ∠Y=90°, the measure of ∠W=60°, and YW = 40 feet. Find the length of WX to the nearest tenth of a foot.

Answers

Answer:

XW = 80 feet

Step-by-step explanation:

To answer this question, we shall be needing a diagrammatic representation of the triangle.

Please check the attachment for this.

We are told to calculate the length XW and this refers to the hypotenuse of the triangle.

The other side that we have is the adjacent.

So yo calculate the length of XW, we need a trigonometric identity that links hypotenuse and adjacent together.

The trigonometric identity to use here is the cosine.

Mathematically;

Cos of an angle = length of the adjacent/length of the hypotenuse

Cos 60 = 40/XW

XW = 40/Cos 60

Cos 60 = 1/2 or 0.5

XW = 40/0.5 = 80 feet

Solve for w.

14 + 3(–w + 6) = –11w

w =

Answers

Answer:

w = -4

Step-by-step explanation:

→Distribute the 3 to (-w + 6):

14 + 3(-w + 6) = -11w

14 -3w + 18 = -11w

→Add like terms:

-3w + 32 = -11w

→Add 11w to both sides:

8w + 32 = 0

→Subtract 32 from both sides:

8w = -32

→Divide both sides by 8

w = -4

Answer:

w= -4

work:

14-3w+18=-11w

32-3w=-11

-3w+11w=-32

8w=-32 divide by 8 then you will get your answer

w=-4

find the perimeter of this figure to the nearest hundredth use 3.14 to approximate pi P=?ft

Answers

Answer:

Step-by-step explanation:

perimeter of the figure = perimeter of the rectangle + perimeter of the semicircle

= 2(l+B) + [tex]\pi[/tex] r

= 2( 10+8) + 3.14 × 5

= 36 + 15.7

= 51.7

= 100 ( rounding off to nearest hundredth )

hope this helps

plz mark it as brailiest!!!!!!

Answer:

48.57 ft

Step-by-step explanation:

Perimeter of Rectangle:

=> 2L+2B

Where L is length and B is Breadth

=> 2(10)+2(8)

=> 20+16

=> 36 ft

Here, Perimeter of Semi-circle:

=> πD/2

Where D = 8

=> (3.14)(8)/2

=> 12.57 ft

Perimeter of the figure

=> 36+12.57 = 48.57 ft

Answer these please!!!!

Answers

Answer:

2/3

Step-by-step explanation:

i hope this helps but um... 2/1÷2/3=2/3

(CO 3) Ten rugby balls are randomly selected from the production line to see if their shape is correct. Over time, the company has found that 89.4% of all their rugby balls have the correct shape. If exactly 7 of the 10 have the right shape, should the company stop the production line? No, as the probability of seven having the correct shape is unusual Yes, as the probability of seven having the correct shape is unusual Yes, as the probability of seven having the correct shape is not unusual No, as the probability of seven having the correct shape is not unusual

Answers

Answer:

Step-by-step explanation:

This is a binomial probability distribution and the probability of success and value is the same for each trial.

Over time, the company has found that 89.4% of all their rugby balls have the correct shape. It means that the probability of success is

89.4/100 = 0.894

If exactly 7 of the 10 have the right shape, should the company stop the production line, it means that the probability of getting right shaped tires in this case is 7/10 = 0.7

This is actually below the known probability of success. Therefore, the correct option is

Yes, as the probability of seven having the correct shape is unusual

Other Questions
Can anyone help please? Larned Corporation recorded the following transactions for the just completed month. $86,000 in raw materials were purchased on account. $84,000 in raw materials were used in production. Of this amount, $70,000 was for direct materials and the remainder was for indirect materials. Total labor wages of $120,000 were paid in cash. Of this amount, $102,100 was for direct labor and the remainder was for indirect labor. Depreciation of $190,000 was incurred on factory equipment. Required: Record the above transactions in journal entries. Help please There are indications that Jem is growing up and growing away from Scout.TrueFalse Please help me!New technology has allowed us to genetically modify our food. Scientist can now create crops that are pest resistant, drought resistant, faster growing and more nutritious. These consequences would best be described asA)positive, intended consequences.B)negative, intended consequences.C)positive, unintended consequences.D)negative, unintended consequences. what is the rate of change between (29,9) and (33,10)? A scatter plot and a possible line of best fit is shown: Is the line of best fit accurate for the data shown? No, because the line does not touch any points No, because the line should touch every point Yes, because it touches the y-axis Yes, because it passes through the center of the data points How is Argentina ranked among the largest countries of the world? A line in the xy-plane passes through the points (1,-2) And has a slope of 1/3. Which of the following points lies on the line?A) (3,-2)B) (2,-4/3)C) (0,-2)D) (-1,-8/3) What is the mechanical advantage of a pulley with 3 support ropes? A large container of water is sitting on a balance. The weight of the container plus the water is 10 newtons. An object the weight of 7 newtons hanging from a forcemeter is lowered towards the surface of the water . what reading on the force meter would you expect? Researchers are interested in determining whether more women than men prefer the beach to the mountains. In a random sample of 200 women, 45% prefer the beach, whereas in a random sample of 300 men, 52% prefer the beach. What is the 99% confidence interval estimate for the difference between the percentages of women and men who prefer the beach over the mountains? Plz help asap, shdhhshdhshxh g Your savings account earns 1.72% interest. You have $3,000 left over from an internship that you will put into the saving account. There is currently no money in the account :( How many years is it until there is $3,756 in the account? Explain why paying money to the government officials to get water is an undemocratic method.(Point stealers will be reported so better dont do it.) ASAP! GIVING BRAINLIEST! Please read the question THEN answer CORRECTLY! NO guessing. I say no guessing because people usually guess on my questions. What is the value of 1 over 2 x+3.4y when x=3 and y = 4? Which of the following tests include 2 components of health-related fitness when tested? a Mile-run. b PACER c Curl-up. d Push-up. e Planks. f Trunk-lift. Piensa en una historia que quieras escribir. Primero, plantea una virtud o un defecto humano que quieras resaltar. ---[recuerda que las fabulas contienen un mensaje morarilizante what is a single word answer to say _I am_ in spanish? 1The use of a wholesale warehouse in the chain of distribution [QAenables direct selling from manufacturer to retailer.B.enables production to be ahead of demand.C. reduces the chain of distribution.Direduces the cost of goods to consumers.